Gaussian Integral: Converting from Cartesian to Polar

Click For Summary
The discussion focuses on converting the Gaussian integral from Cartesian to polar coordinates. The integral ##\int_{-\infty}^{\infty} e^{-x^2}dx## is evaluated by recognizing that its square can be expressed as a double integral over the area in Cartesian coordinates, which transitions to polar coordinates with the area element changing from ##dx \, dy## to ##r \, dr \, d\theta##. The conversion involves understanding the relationships between Cartesian coordinates (x, y) and polar coordinates (r, θ), particularly how to express the differential area and the relationships between the variables. The discussion highlights the importance of Jacobians in these transformations, which are typically covered in Calculus II. Overall, the thread emphasizes the mathematical principles underlying the conversion process.
Adoniram
Messages
93
Reaction score
6

Homework Statement


I'm encountering these integrals a lot lately, and I can solve them because I know the "trick" but I'd like to know actually how the cartesian to polar conversion works:
##\int_{-\infty}^{\infty}e^{-x^2}dx##

Homework Equations


##\int_{-\infty}^{\infty} e^{-x^2} = I##
##I^2=\int_{-\infty}^{\infty} e^{-x^2}dx \int_{-\infty}^{\infty} e^{-y^2}dy##
##=\int_{-\infty}^{\infty} e^{-r^2}r dr dθ = π##

The Attempt at a Solution


So, if I look at ##r=\sqrt{x^2+y^2}##, it's easy to see that ##dr=(1/r)(x dx+y dy)##
Which leads me to believe that ##dθ=(x dx+y dy)## ... ?

If ##θ=ArcTan(y/x)##, then how does ##dθ=(x dx+y dy)##?

Say ##y=Tan(θ)x##, then taking ##dy## and simplifying, I can get:
##x dy - y dx=(x^2+y^2) dθ##

So unless ##x/(x^2+y^2) = y## and ##y/(x^2+y^2) = -x##, I am at a loss... :(
 
Physics news on Phys.org
Adoniram said:

Homework Statement


I'm encountering these integrals a lot lately, and I can solve them because I know the "trick" but I'd like to know actually how the cartesian to polar conversion works:
##\int_{-\infty}^{\infty}e^{-x^2}dx##

Homework Equations


##\int_{-\infty}^{\infty} e^{-x^2} = I##
##I^2=\int_{-\infty}^{\infty} e^{-x^2}dx \int_{-\infty}^{\infty} e^{-y^2}dy##
##=\int_{-\infty}^{\infty} e^{-r^2}r dr dθ = π##

The Attempt at a Solution


So, if I look at ##r=\sqrt{x^2+y^2}##, it's easy to see that ##dr=(1/r)(x dx+y dy)##
Which leads me to believe that ##dθ=(x dx+y dy)## ... ?

If ##θ=ArcTan(y/x)##, then how does ##dθ=(x dx+y dy)##?

Say ##y=Tan(θ)x##, then taking ##dy## and simplifying, I can get:
##x dy - y dx=(x^2+y^2) dθ##

So unless ##x/(x^2+y^2) = y## and ##y/(x^2+y^2) = -x##, I am at a loss... :(

We can write
I^2 = \int_{-\infty}^{\infty} e^{-x^2}dx \int_{-\infty}^{\infty} e^{-y^2}dy = \int_{R^2} e^{-x^2-y^2} \, dA,
where ##dA = dx dy## is the two-dimensional "area element". When we switch to polar coordinates, the area element becomes ##dA = r \, dr \, d\theta##. Basically, we have ## I^2 \approx \sum_i \Delta A_i e^{-r^2_i} ##, where the ##\Delta A_1, \Delta A_2, \ldots ## are finite but small areas of little regions and the ##r_i## are the ##\sqrt{x_i^2 + y_i^2}## values at some point ##(x_i,y_i)## inside the regions ##i = 1,2, \ldots##. We can either take the little regions to be rectangular, with horizontal and vertical sides ##\Delta x_i## and ##\Delta y_i##, or else we can take them to be slices of radial wedges with sides along radial lines at angles ##\theta_i## and ##\theta_i + \Delta \theta_i## and inner-outer radii at ##r_i## and ##r_i + \Delta r_i##. In rectangular coordinates the area element is ##\Delta A_i = \Delta x_i \, \Delta y_i##, while in polar coordinates it is ##\Delta A_i = r_i \, \Delta r_i \, \Delta \theta_i##.

All this was supposed to have been covered thoroughly in Calculus II, where things such as Jacobians and the like were introduced and motivated.
 
  • Like
Likes RUber
Question: A clock's minute hand has length 4 and its hour hand has length 3. What is the distance between the tips at the moment when it is increasing most rapidly?(Putnam Exam Question) Answer: Making assumption that both the hands moves at constant angular velocities, the answer is ## \sqrt{7} .## But don't you think this assumption is somewhat doubtful and wrong?

Similar threads

Replies
4
Views
2K
  • · Replies 7 ·
Replies
7
Views
2K
Replies
2
Views
2K
  • · Replies 10 ·
Replies
10
Views
2K
  • · Replies 47 ·
2
Replies
47
Views
4K
Replies
6
Views
2K
  • · Replies 19 ·
Replies
19
Views
2K
  • · Replies 9 ·
Replies
9
Views
2K
Replies
9
Views
2K
  • · Replies 14 ·
Replies
14
Views
2K